If $limlimits_{z to infty} |F(z)|=0$, can $limlimits_{z to infty} F(z) ne 0$? Announcing the...

Why is there so little support for joining EFTA in the British parliament?

First paper to introduce the "principal-agent problem"

The Nth Gryphon Number

An isoperimetric-type inequality inside a cube

Table formatting with tabularx?

Can two people see the same photon?

Weaponising the Grasp-at-a-Distance spell

My mentor says to set image to Fine instead of RAW — how is this different from JPG?

Why can't fire hurt Daenerys but it did to Jon Snow in season 1?

Why are two-digit numbers in Jonathan Swift's "Gulliver's Travels" (1726) written in "German style"?

Does a random sequence of vectors span a Hilbert space?

Why complex landing gears are used instead of simple, reliable and light weight muscle wire or shape memory alloys?

Where did Ptolemy compare the Earth to the distance of fixed stars?

calculator's angle answer for trig ratios that can work in more than 1 quadrant on the unit circle

Was the pager message from Nick Fury to Captain Marvel unnecessary?

Does the universe have a fixed centre of mass?

.bashrc alias for a command with fixed second parameter

Russian equivalents of おしゃれは足元から (Every good outfit starts with the shoes)

Plotting a Maclaurin series

Can I cut the hair of a conjured korred with a blade made of precious material to harvest that material from the korred?

What should one know about term logic before studying propositional and predicate logic?

Noise in Eigenvalues plot

Dinosaur Word Search, Letter Solve, and Unscramble

By what mechanism was the 2017 UK General Election called?



If $limlimits_{z to infty} |F(z)|=0$, can $limlimits_{z to infty} F(z) ne 0$?



Announcing the arrival of Valued Associate #679: Cesar Manara
Planned maintenance scheduled April 23, 2019 at 23:30 UTC (7:30pm US/Eastern)Finding $lim limits_{zto i} frac{1}{(z-i)^2}$ rigorously$limlimits_{xto-infty}sqrt{4x^2+3x}+2x$How can I find $lim limits_{nto infty} frac{ln(q_{n+1})}{ln(q_n)}$?Prove that if $lim limits_{n to infty}$ $x_n$ = $L$, then $lim limits_{n to infty}$ $|x_n|$ = $|L|$.Does the existence of $mathop {lim }limits_{x to + infty } frac{{f(x)}}{x}$ always imply the existence of asymptote?If $f$ is convex (or concave), does the existence of $mathop {lim }limits_{x to + infty } frac{{f(x)}}{x}$ imply the existence of asymptote?Show that if $limlimits_{x to infty} f(x)$ exists and $f''$ is bounded, then $limlimits_{x to infty} f'(x)=0$.$sum u_n$ converges $implies$ $lim_{n to infty} n u_n = 0$.Solving Limits: Why must I multiply by Conjugate? $lim limits_{n to infty}$ √(n+1) - √nIs this way of finding $limlimits_{xto +infty}(x-ln(x^2+1))$ valid?












1












$begingroup$


To prove the existence of limit of $F(z)$ at $z=infty$ in here (Eq. 1.8 page 3) it says that since $limlimits_{z to infty} |F(z)|=0$ thus $limlimits_{z to infty} F(z)=w_n$ exists; Of course exits, but also must $w_n=0$. But Nor this article neither the book Churchill Sec 117 Exercise 2 doesn't say $w_n=0$. Can $w_n ne 0$ be possible?










share|cite|improve this question











$endgroup$

















    1












    $begingroup$


    To prove the existence of limit of $F(z)$ at $z=infty$ in here (Eq. 1.8 page 3) it says that since $limlimits_{z to infty} |F(z)|=0$ thus $limlimits_{z to infty} F(z)=w_n$ exists; Of course exits, but also must $w_n=0$. But Nor this article neither the book Churchill Sec 117 Exercise 2 doesn't say $w_n=0$. Can $w_n ne 0$ be possible?










    share|cite|improve this question











    $endgroup$















      1












      1








      1





      $begingroup$


      To prove the existence of limit of $F(z)$ at $z=infty$ in here (Eq. 1.8 page 3) it says that since $limlimits_{z to infty} |F(z)|=0$ thus $limlimits_{z to infty} F(z)=w_n$ exists; Of course exits, but also must $w_n=0$. But Nor this article neither the book Churchill Sec 117 Exercise 2 doesn't say $w_n=0$. Can $w_n ne 0$ be possible?










      share|cite|improve this question











      $endgroup$




      To prove the existence of limit of $F(z)$ at $z=infty$ in here (Eq. 1.8 page 3) it says that since $limlimits_{z to infty} |F(z)|=0$ thus $limlimits_{z to infty} F(z)=w_n$ exists; Of course exits, but also must $w_n=0$. But Nor this article neither the book Churchill Sec 117 Exercise 2 doesn't say $w_n=0$. Can $w_n ne 0$ be possible?







      calculus complex-analysis limits proof-explanation






      share|cite|improve this question















      share|cite|improve this question













      share|cite|improve this question




      share|cite|improve this question








      edited Mar 26 at 11:08









      rash

      568216




      568216










      asked Mar 26 at 11:02









      72D72D

      298117




      298117






















          1 Answer
          1






          active

          oldest

          votes


















          1












          $begingroup$

          The limit must be zero. Suppose $lim F(z)=w_n$ exists. Then $lim | F(z)-w_n |=0$. However $|F(z)-w_n| geq big||F(z)|-|w_n| big|$, for any $z$ by the triangle inequality, so it follows that $lim big||F(z)|-|w_n| big|=0$. That is, $lim |F(z)|=|w_n|$.



          Note that the convergence of $|F(z)|$ does not imply $F(z)$ converges unless $lim |F(z)|=0$.






          share|cite|improve this answer









          $endgroup$













          • $begingroup$
            $w_n=F(infty)$ for Schwarz-Christoffel transformations which is not zero always. That's why I am confused!
            $endgroup$
            – 72D
            Mar 26 at 11:19










          • $begingroup$
            Honestly I think the document you linked is not terribly well written.The appearance of individual equations is messy, the spacing is all over the place, the alignment is bad. e.g. in equation 1.6 half of the equation is centered and half is not. The exponents are very far from the terms they exponetiate. Work like this is hard to follow, both for the reader and the author, and it leaves room for the author to have made a mistake. In equatioin 1.8 the author bounds the limit of $F(z)$ above by zero and uses that to conclude that $F(z)$ converges to some value.
            $endgroup$
            – Eric
            Mar 26 at 12:09










          • $begingroup$
            There are two problems there. First, the author hasn't shown that the limit has a lower bound, so from what is shown they cannot conclude a limit exists. Now the equations can easily be applied to $-F(z)$ to get that the limit is bounded below by zero, but they should have at least commented on this. Now if the limit is both bounded above and below by zero then the limit must be zero. So why do they say it converges to some value and not emphasize zero? I didn't carefully check the details in the document, but I wouldn't trust what is written in without checking extremely carefully.
            $endgroup$
            – Eric
            Mar 26 at 12:12










          • $begingroup$
            I also found that the journal which published the article you linked is on the predatory journal list found at predatoryjournals.com/journals/#I. In short, you should not trust that article or anything from that journal.
            $endgroup$
            – Eric
            Mar 26 at 12:21














          Your Answer








          StackExchange.ready(function() {
          var channelOptions = {
          tags: "".split(" "),
          id: "69"
          };
          initTagRenderer("".split(" "), "".split(" "), channelOptions);

          StackExchange.using("externalEditor", function() {
          // Have to fire editor after snippets, if snippets enabled
          if (StackExchange.settings.snippets.snippetsEnabled) {
          StackExchange.using("snippets", function() {
          createEditor();
          });
          }
          else {
          createEditor();
          }
          });

          function createEditor() {
          StackExchange.prepareEditor({
          heartbeatType: 'answer',
          autoActivateHeartbeat: false,
          convertImagesToLinks: true,
          noModals: true,
          showLowRepImageUploadWarning: true,
          reputationToPostImages: 10,
          bindNavPrevention: true,
          postfix: "",
          imageUploader: {
          brandingHtml: "Powered by u003ca class="icon-imgur-white" href="https://imgur.com/"u003eu003c/au003e",
          contentPolicyHtml: "User contributions licensed under u003ca href="https://creativecommons.org/licenses/by-sa/3.0/"u003ecc by-sa 3.0 with attribution requiredu003c/au003e u003ca href="https://stackoverflow.com/legal/content-policy"u003e(content policy)u003c/au003e",
          allowUrls: true
          },
          noCode: true, onDemand: true,
          discardSelector: ".discard-answer"
          ,immediatelyShowMarkdownHelp:true
          });


          }
          });














          draft saved

          draft discarded


















          StackExchange.ready(
          function () {
          StackExchange.openid.initPostLogin('.new-post-login', 'https%3a%2f%2fmath.stackexchange.com%2fquestions%2f3163033%2fif-lim-limits-z-to-infty-fz-0-can-lim-limits-z-to-infty-fz%23new-answer', 'question_page');
          }
          );

          Post as a guest















          Required, but never shown

























          1 Answer
          1






          active

          oldest

          votes








          1 Answer
          1






          active

          oldest

          votes









          active

          oldest

          votes






          active

          oldest

          votes









          1












          $begingroup$

          The limit must be zero. Suppose $lim F(z)=w_n$ exists. Then $lim | F(z)-w_n |=0$. However $|F(z)-w_n| geq big||F(z)|-|w_n| big|$, for any $z$ by the triangle inequality, so it follows that $lim big||F(z)|-|w_n| big|=0$. That is, $lim |F(z)|=|w_n|$.



          Note that the convergence of $|F(z)|$ does not imply $F(z)$ converges unless $lim |F(z)|=0$.






          share|cite|improve this answer









          $endgroup$













          • $begingroup$
            $w_n=F(infty)$ for Schwarz-Christoffel transformations which is not zero always. That's why I am confused!
            $endgroup$
            – 72D
            Mar 26 at 11:19










          • $begingroup$
            Honestly I think the document you linked is not terribly well written.The appearance of individual equations is messy, the spacing is all over the place, the alignment is bad. e.g. in equation 1.6 half of the equation is centered and half is not. The exponents are very far from the terms they exponetiate. Work like this is hard to follow, both for the reader and the author, and it leaves room for the author to have made a mistake. In equatioin 1.8 the author bounds the limit of $F(z)$ above by zero and uses that to conclude that $F(z)$ converges to some value.
            $endgroup$
            – Eric
            Mar 26 at 12:09










          • $begingroup$
            There are two problems there. First, the author hasn't shown that the limit has a lower bound, so from what is shown they cannot conclude a limit exists. Now the equations can easily be applied to $-F(z)$ to get that the limit is bounded below by zero, but they should have at least commented on this. Now if the limit is both bounded above and below by zero then the limit must be zero. So why do they say it converges to some value and not emphasize zero? I didn't carefully check the details in the document, but I wouldn't trust what is written in without checking extremely carefully.
            $endgroup$
            – Eric
            Mar 26 at 12:12










          • $begingroup$
            I also found that the journal which published the article you linked is on the predatory journal list found at predatoryjournals.com/journals/#I. In short, you should not trust that article or anything from that journal.
            $endgroup$
            – Eric
            Mar 26 at 12:21


















          1












          $begingroup$

          The limit must be zero. Suppose $lim F(z)=w_n$ exists. Then $lim | F(z)-w_n |=0$. However $|F(z)-w_n| geq big||F(z)|-|w_n| big|$, for any $z$ by the triangle inequality, so it follows that $lim big||F(z)|-|w_n| big|=0$. That is, $lim |F(z)|=|w_n|$.



          Note that the convergence of $|F(z)|$ does not imply $F(z)$ converges unless $lim |F(z)|=0$.






          share|cite|improve this answer









          $endgroup$













          • $begingroup$
            $w_n=F(infty)$ for Schwarz-Christoffel transformations which is not zero always. That's why I am confused!
            $endgroup$
            – 72D
            Mar 26 at 11:19










          • $begingroup$
            Honestly I think the document you linked is not terribly well written.The appearance of individual equations is messy, the spacing is all over the place, the alignment is bad. e.g. in equation 1.6 half of the equation is centered and half is not. The exponents are very far from the terms they exponetiate. Work like this is hard to follow, both for the reader and the author, and it leaves room for the author to have made a mistake. In equatioin 1.8 the author bounds the limit of $F(z)$ above by zero and uses that to conclude that $F(z)$ converges to some value.
            $endgroup$
            – Eric
            Mar 26 at 12:09










          • $begingroup$
            There are two problems there. First, the author hasn't shown that the limit has a lower bound, so from what is shown they cannot conclude a limit exists. Now the equations can easily be applied to $-F(z)$ to get that the limit is bounded below by zero, but they should have at least commented on this. Now if the limit is both bounded above and below by zero then the limit must be zero. So why do they say it converges to some value and not emphasize zero? I didn't carefully check the details in the document, but I wouldn't trust what is written in without checking extremely carefully.
            $endgroup$
            – Eric
            Mar 26 at 12:12










          • $begingroup$
            I also found that the journal which published the article you linked is on the predatory journal list found at predatoryjournals.com/journals/#I. In short, you should not trust that article or anything from that journal.
            $endgroup$
            – Eric
            Mar 26 at 12:21
















          1












          1








          1





          $begingroup$

          The limit must be zero. Suppose $lim F(z)=w_n$ exists. Then $lim | F(z)-w_n |=0$. However $|F(z)-w_n| geq big||F(z)|-|w_n| big|$, for any $z$ by the triangle inequality, so it follows that $lim big||F(z)|-|w_n| big|=0$. That is, $lim |F(z)|=|w_n|$.



          Note that the convergence of $|F(z)|$ does not imply $F(z)$ converges unless $lim |F(z)|=0$.






          share|cite|improve this answer









          $endgroup$



          The limit must be zero. Suppose $lim F(z)=w_n$ exists. Then $lim | F(z)-w_n |=0$. However $|F(z)-w_n| geq big||F(z)|-|w_n| big|$, for any $z$ by the triangle inequality, so it follows that $lim big||F(z)|-|w_n| big|=0$. That is, $lim |F(z)|=|w_n|$.



          Note that the convergence of $|F(z)|$ does not imply $F(z)$ converges unless $lim |F(z)|=0$.







          share|cite|improve this answer












          share|cite|improve this answer



          share|cite|improve this answer










          answered Mar 26 at 11:17









          EricEric

          37811




          37811












          • $begingroup$
            $w_n=F(infty)$ for Schwarz-Christoffel transformations which is not zero always. That's why I am confused!
            $endgroup$
            – 72D
            Mar 26 at 11:19










          • $begingroup$
            Honestly I think the document you linked is not terribly well written.The appearance of individual equations is messy, the spacing is all over the place, the alignment is bad. e.g. in equation 1.6 half of the equation is centered and half is not. The exponents are very far from the terms they exponetiate. Work like this is hard to follow, both for the reader and the author, and it leaves room for the author to have made a mistake. In equatioin 1.8 the author bounds the limit of $F(z)$ above by zero and uses that to conclude that $F(z)$ converges to some value.
            $endgroup$
            – Eric
            Mar 26 at 12:09










          • $begingroup$
            There are two problems there. First, the author hasn't shown that the limit has a lower bound, so from what is shown they cannot conclude a limit exists. Now the equations can easily be applied to $-F(z)$ to get that the limit is bounded below by zero, but they should have at least commented on this. Now if the limit is both bounded above and below by zero then the limit must be zero. So why do they say it converges to some value and not emphasize zero? I didn't carefully check the details in the document, but I wouldn't trust what is written in without checking extremely carefully.
            $endgroup$
            – Eric
            Mar 26 at 12:12










          • $begingroup$
            I also found that the journal which published the article you linked is on the predatory journal list found at predatoryjournals.com/journals/#I. In short, you should not trust that article or anything from that journal.
            $endgroup$
            – Eric
            Mar 26 at 12:21




















          • $begingroup$
            $w_n=F(infty)$ for Schwarz-Christoffel transformations which is not zero always. That's why I am confused!
            $endgroup$
            – 72D
            Mar 26 at 11:19










          • $begingroup$
            Honestly I think the document you linked is not terribly well written.The appearance of individual equations is messy, the spacing is all over the place, the alignment is bad. e.g. in equation 1.6 half of the equation is centered and half is not. The exponents are very far from the terms they exponetiate. Work like this is hard to follow, both for the reader and the author, and it leaves room for the author to have made a mistake. In equatioin 1.8 the author bounds the limit of $F(z)$ above by zero and uses that to conclude that $F(z)$ converges to some value.
            $endgroup$
            – Eric
            Mar 26 at 12:09










          • $begingroup$
            There are two problems there. First, the author hasn't shown that the limit has a lower bound, so from what is shown they cannot conclude a limit exists. Now the equations can easily be applied to $-F(z)$ to get that the limit is bounded below by zero, but they should have at least commented on this. Now if the limit is both bounded above and below by zero then the limit must be zero. So why do they say it converges to some value and not emphasize zero? I didn't carefully check the details in the document, but I wouldn't trust what is written in without checking extremely carefully.
            $endgroup$
            – Eric
            Mar 26 at 12:12










          • $begingroup$
            I also found that the journal which published the article you linked is on the predatory journal list found at predatoryjournals.com/journals/#I. In short, you should not trust that article or anything from that journal.
            $endgroup$
            – Eric
            Mar 26 at 12:21


















          $begingroup$
          $w_n=F(infty)$ for Schwarz-Christoffel transformations which is not zero always. That's why I am confused!
          $endgroup$
          – 72D
          Mar 26 at 11:19




          $begingroup$
          $w_n=F(infty)$ for Schwarz-Christoffel transformations which is not zero always. That's why I am confused!
          $endgroup$
          – 72D
          Mar 26 at 11:19












          $begingroup$
          Honestly I think the document you linked is not terribly well written.The appearance of individual equations is messy, the spacing is all over the place, the alignment is bad. e.g. in equation 1.6 half of the equation is centered and half is not. The exponents are very far from the terms they exponetiate. Work like this is hard to follow, both for the reader and the author, and it leaves room for the author to have made a mistake. In equatioin 1.8 the author bounds the limit of $F(z)$ above by zero and uses that to conclude that $F(z)$ converges to some value.
          $endgroup$
          – Eric
          Mar 26 at 12:09




          $begingroup$
          Honestly I think the document you linked is not terribly well written.The appearance of individual equations is messy, the spacing is all over the place, the alignment is bad. e.g. in equation 1.6 half of the equation is centered and half is not. The exponents are very far from the terms they exponetiate. Work like this is hard to follow, both for the reader and the author, and it leaves room for the author to have made a mistake. In equatioin 1.8 the author bounds the limit of $F(z)$ above by zero and uses that to conclude that $F(z)$ converges to some value.
          $endgroup$
          – Eric
          Mar 26 at 12:09












          $begingroup$
          There are two problems there. First, the author hasn't shown that the limit has a lower bound, so from what is shown they cannot conclude a limit exists. Now the equations can easily be applied to $-F(z)$ to get that the limit is bounded below by zero, but they should have at least commented on this. Now if the limit is both bounded above and below by zero then the limit must be zero. So why do they say it converges to some value and not emphasize zero? I didn't carefully check the details in the document, but I wouldn't trust what is written in without checking extremely carefully.
          $endgroup$
          – Eric
          Mar 26 at 12:12




          $begingroup$
          There are two problems there. First, the author hasn't shown that the limit has a lower bound, so from what is shown they cannot conclude a limit exists. Now the equations can easily be applied to $-F(z)$ to get that the limit is bounded below by zero, but they should have at least commented on this. Now if the limit is both bounded above and below by zero then the limit must be zero. So why do they say it converges to some value and not emphasize zero? I didn't carefully check the details in the document, but I wouldn't trust what is written in without checking extremely carefully.
          $endgroup$
          – Eric
          Mar 26 at 12:12












          $begingroup$
          I also found that the journal which published the article you linked is on the predatory journal list found at predatoryjournals.com/journals/#I. In short, you should not trust that article or anything from that journal.
          $endgroup$
          – Eric
          Mar 26 at 12:21






          $begingroup$
          I also found that the journal which published the article you linked is on the predatory journal list found at predatoryjournals.com/journals/#I. In short, you should not trust that article or anything from that journal.
          $endgroup$
          – Eric
          Mar 26 at 12:21




















          draft saved

          draft discarded




















































          Thanks for contributing an answer to Mathematics Stack Exchange!


          • Please be sure to answer the question. Provide details and share your research!

          But avoid



          • Asking for help, clarification, or responding to other answers.

          • Making statements based on opinion; back them up with references or personal experience.


          Use MathJax to format equations. MathJax reference.


          To learn more, see our tips on writing great answers.




          draft saved


          draft discarded














          StackExchange.ready(
          function () {
          StackExchange.openid.initPostLogin('.new-post-login', 'https%3a%2f%2fmath.stackexchange.com%2fquestions%2f3163033%2fif-lim-limits-z-to-infty-fz-0-can-lim-limits-z-to-infty-fz%23new-answer', 'question_page');
          }
          );

          Post as a guest















          Required, but never shown





















































          Required, but never shown














          Required, but never shown












          Required, but never shown







          Required, but never shown

































          Required, but never shown














          Required, but never shown












          Required, but never shown







          Required, but never shown







          Popular posts from this blog

          Magento 2 - Add success message with knockout Planned maintenance scheduled April 23, 2019 at 23:30 UTC (7:30pm US/Eastern) Announcing the arrival of Valued Associate #679: Cesar Manara Unicorn Meta Zoo #1: Why another podcast?Success / Error message on ajax request$.widget is not a function when loading a homepage after add custom jQuery on custom themeHow can bind jQuery to current document in Magento 2 When template load by ajaxRedirect page using plugin in Magento 2Magento 2 - Update quantity and totals of cart page without page reload?Magento 2: Quote data not loaded on knockout checkoutMagento 2 : I need to change add to cart success message after adding product into cart through pluginMagento 2.2.5 How to add additional products to cart from new checkout step?Magento 2 Add error/success message with knockoutCan't validate Post Code on checkout page

          Fil:Tokke komm.svg

          Where did Arya get these scars? Unicorn Meta Zoo #1: Why another podcast? Announcing the arrival of Valued Associate #679: Cesar Manara Favourite questions and answers from the 1st quarter of 2019Why did Arya refuse to end it?Has the pronunciation of Arya Stark's name changed?Has Arya forgiven people?Why did Arya Stark lose her vision?Why can Arya still use the faces?Has the Narrow Sea become narrower?Does Arya Stark know how to make poisons outside of the House of Black and White?Why did Nymeria leave Arya?Why did Arya not kill the Lannister soldiers she encountered in the Riverlands?What is the current canonical age of Sansa, Bran and Arya Stark?